[exo] chaises et bénéfice
Répondre à la discussion
Affichage des résultats 1 à 20 sur 20

[exo] chaises et bénéfice



  1. #1
    invite37e81c9f

    [exo] chaises et bénéfice


    ------

    Bonjours je m'appelle anne-sophie
    j'ai un problème de maths à résoudre mais je ne comprends pas.
    Je vous demande votre aide.
    Merci.
    Voilà l'énoncée:
    une petite entreprise fabrique et vend des chaises.Le coût de fabrication en euros de q chaises est donné par : C(q)=0,1q(au carré) +4q+1000.
    Chaque chaises est vendue 29 euros.
    1) Par quels nombres de chaises fabriquées le coût est-il inférieurs à 2000 euros?
    2) a. Exprimer le bénéfice B(q) en fonction de q.
    b. Pour quels nombres de chaises fabriquées et vendues ce bénéfice est-il positif ou nul?


    Merci encore.

    Au revoir.

    -----

  2. #2
    Seirios

    Re : bonjours

    Bonjour,

    1) Par quels nombres de chaises fabriquées le coût est-il inférieurs à 2000 euros?
    Il suffit de résoudre C(q)<2000

    2) a. Exprimer le bénéfice B(q) en fonction de q.
    Tu as le bénéfice, et donc B(q), qui la différence entre le prix de vente et le coût de vente.

    b. Pour quels nombres de chaises fabriquées et vendues ce bénéfice est-il positif ou nul?
    Une fois que tu as répondu à la question 2)a. tu as une expression du bénéfice, et tu résous B(q)>=0.
    If your method does not solve the problem, change the problem.

  3. #3
    invite37e81c9f

    Re : bonjours

    Merci de me répondre.
    Pour la question 1, je trouve 2 solution car j'utile delta je trouve donc
    -4+ racine 416/0.2 et -4+racine 416/0.2


    Mais je ne comprends pas si il faut faire un tableau de signe ou si se sont ces deux résultat qui est le nombre de chaises fabriquées lorsque le coût est inférieur à 2000 euros
    et si ce sont les résultat je ne voit pas commente on peut l'expliquer en fontion du bénéfice?

  4. #4
    Seirios

    Re : bonjours

    Il faut bien faire un tableau de signe. Une fois le tableau de signe de C(q)-2000 de dressé, on saura lorsque C(q)-2000<0 et quand C(q)-2000>0, c'est-à-dire lorsque C(q)<2000 et C(q)>2000.
    If your method does not solve the problem, change the problem.

  5. A voir en vidéo sur Futura
  6. #5
    invite37e81c9f

    Re : bonjours

    j'ai trouvé
    S = )-4-V416/0.2; -4+V416/0.2(

    Mais sa veut dire que tout les nombre compris entre -4-V416/0.2 et -4+V416/0.2 le coût de fabrication est inférieur à 2000 euros?

  7. #6
    invite37e81c9f

    Re : bonjours

    Mais on ne peut pas avoir un nombre de chaises négatif .
    Je ne comprends vraiment pas.

  8. #7
    Seirios

    Re : bonjours

    Mais on ne peut pas avoir un nombre de chaises négatif .
    Exactement, donc ton tableau de signe se fera sur [0,+oo[. Par conséquent, ton résultat :

    j'ai trouvé
    S = )-4-V416/0.2; -4+V416/0.2(
    est juste d'un point de vue purement mathématique, mais pour garder une certaine cohérence avec l'énoncé, tu ne garderas que la partie positive.
    If your method does not solve the problem, change the problem.

  9. #8
    invite37e81c9f

    Re : bonjours

    donc la réponse de la question 1 est )0; -4+V416/0.2(
    Mais je ne vois pas commente on peut expliquer ce résultat en fonction du bénéfice comme ce sont des intervalles .

  10. #9
    invitec7217a00

    Re : bonjours

    Bonjour,
    est-ce que tu as bien vérifié que tes racines sont correctes?
    vérifies si , sinon c'est que tu dois refaire ton calcul

  11. #10
    Seirios

    Re : bonjours

    donc la réponse de la question 1 est )0; -4+V416/0.2(
    C'est bien ça, mais je pense que pour rester encore davantage en cohérence avec l'énoncé, tu peux te permettre d'arrondir ta borne supérieur à l'unité supérieure.

    Mais je ne vois pas commente on peut expliquer ce résultat en fonction du bénéfice comme ce sont des intervalles
    Mais tu n'as pas besoin...C'est la réponse à la question 1, et elle totalement séparée de la question 2 (si j'ai bien saisi ta question).
    If your method does not solve the problem, change the problem.

  12. #11
    invite37e81c9f

    Re : bonjours

    donc si je comprends bien pour exprimer le bénéfice B(q) en fonction de q
    il faut
    b(q) - 0.1q+4q+1000

  13. #12
    Seirios

    Re : bonjours

    donc si je comprends bien pour exprimer le bénéfice B(q) en fonction de q
    il faut
    b(q) - 0.1q+4q+1000
    Que représente b(q) ? Normalement, le bénéfice représente le prix de vente pour q chaises (soit 29q) moins le coût de production pour q chaises (soit C(q)). C'est tout simplement l'argent que gagne le constructeur.
    If your method does not solve the problem, change the problem.

  14. #13
    invite37e81c9f

    Re : bonjours

    donc B(q) : 29q - (0.1q au carré +4q+1000)

  15. #14
    invite37e81c9f

    Re : bonjours

    si je fais B(q) supérieur ou égale à 0

    j'obtient

    (0;50)U(200;+infini(

  16. #15
    Seirios

    Re : bonjours

    donc B(q) : 29q - (0.1q au carré +4q+1000)
    C'est bien ça.

    si je fais B(q) supérieur ou égale à 0

    j'obtient

    (0;50)U(200;+infini(
    Quel est le polynôme que tu obtiens ? Quel est son discriminant ? (parce que je trouve un résultat différent...)
    If your method does not solve the problem, change the problem.

  17. #16
    invite37e81c9f

    Re : bonjours

    j'obtient delta = 225
    donc 2 solutions
    50 et 200
    donc je met le signe de a à l'extérieur des racines
    et x est toujours positif .

  18. #17
    invite37e81c9f

    Re : bonjours

    oui mon résultat est faux
    quand je prends le nombre de chaises =20
    le bénéfice est négatif or il faut qu'il soit positif ou nul
    mai je ne voit pas où j'aurais pu faire une erreur de calcul.

  19. #18
    Seirios

    Re : bonjours

    j'obtient delta = 225
    donc 2 solutions
    50 et 200
    Oui c'est bien ça, j'ai fait une erreur de calcul...

    donc je met le signe de a à l'extérieur des racines.
    Sauf que dans ce cas, a est négatif, donc B(q) sera positif à l'intérieur de ses racines.
    If your method does not solve the problem, change the problem.

  20. #19
    invite37e81c9f

    Re : bonjours

    pourtant on a 29q et pa -29q
    je comprends pas pourquoi a est négatif sinon tout les calcul sont faux

  21. #20
    invite37e81c9f

    Re : bonjours

    ah oui j'ai confondu a et b
    d'acord j'ai compris
    merci beaucoup de m'avoir dépondu et de m'avoir expliqué.

Discussions similaires

  1. Etudier le bénéfice d'une entreprise
    Par invite53ccff2f dans le forum Mathématiques du collège et du lycée
    Réponses: 8
    Dernier message: 08/09/2007, 20h22
  2. Bénéfice et approximation
    Par inviteb23e3eef dans le forum Mathématiques du collège et du lycée
    Réponses: 1
    Dernier message: 01/11/2006, 12h38
  3. exo de sup!
    Par invite7e62803d dans le forum Mathématiques du supérieur
    Réponses: 7
    Dernier message: 29/10/2004, 15h28